CCRN Practice Questions - Cardiovascular

Lakukan tugas rumah & ujian kamu dengan baik sekarang menggunakan Quizwiz!

An S4 is an expected physical finding in which of the following? a. Acute myocardial infarction b. Left ventricular failure c. Pericarditis d. Bundle branch block

Correct answer: a Rationale: An S4 is an indication of ventricular noncompliance. Noncompliance is caused by ischemia, infarction, hypertrophy, cardiac tamponade, constrictive pericarditis, or restrictive cardiomyopathy. Left ventricular failure would cause an S3. Pericarditis would cause a pericardial friction rub. Bundle branch blocks cause splits (left bundle branch block causes a paradoxical split of S2, and right bundle branch block causes a split of S1 and increased splitting of S2 during inspiration). Test-Taking Strategy: Read the question carefully and note the key word expected. Associate S4 with a noncompliant ventricle. Ischemia, infarction, and hypertrophy cause noncompliance. Choose option a.

A patient is admitted with unstable angina. He has a long history of hypertension and coronary artery disease. The nurse notes a split S2 on expiration and a single S2 on inspiration during cardiac auscultation. Blood pressure is 150/88 mm Hg, and heart rate is 88 beats/min. On the electrocardiogram, there is a normal-appearing P wave in front of each QRS complex, the PR interval measures 0.2 second consistently, and the QRS complexes measure 0.14 second. They are positive in V5 and V6 and negative in V1 and V2. These findings most likely indicate which of the following? a. Left bundle branch block (LBBB) b. Right bundle branch block (RBBB) c. Third-degree atrioventricular block d. Ventricular tachycardia

Correct answer: a Rationale: Features of LBBB described here are a QRS complex greater than 0.12 second in duration and a QRS complex that is positive in leads V5 and V6 (consider these left ventricular leads) and negative in leads V1 and V2 (consider these right ventricular leads). LBBB causes a paradoxical splitting of S2. This means that it is split on expiration but not on inspiration. This is paradoxical because it is opposite of a normal physiologic split of S2, which is split on inspiration but not split on expiration. Test-Taking Strategy: A P wave precedes each QRS complex, so eliminate ventricular tachycardia, option d. A P wave occurs for each QRS complex, and the PR interval is consistent, so eliminate third-degree atrioventricular block, option c. The problem is a bundle branch block because it originates above the ventricle (note normal and consistent P waves), but the QRS complex is wide, indicating that the impulse is taking longer than normal to travel through the ventricle. Is the block left or right? Remember that the wide QRS complex will be upright in leads V1 and V2 if it is RBBB but upright in leads V5 and V6 if it is LBBB. Also, in looking at lead V1, if there is an associated P wave but the QRS complex is wide and upright, it is RBBB. If the QRS complex is wide and negative, it is LBBB. Choose option a.

A 52-year-old man has undergone coronary artery bypass grafting and has returned to the critical care unit. During the last hour his parameters were as follows: O2 saturation 99% PaO2 with FiO2 0.4 184 mm Hg Hemoglobin 12 gm/dL Serum potassium 3.8 mEq/L Urine output 100 mL/hr Now the patient's venous oxygen saturation (SvO2) monitor shows a rapid decline from a reading of 75% to 62% without any change in arterial oxygen saturation (SaO2). His cardiac rhythm changes from normal sinus rhythm to a sinus rhythm with ventricular bigeminy. The nurse would assess the patient for which of the following? a. Decreased cardiac output b. Internal hemorrhage c. Hypermetabolism d. Hypoxemia

Correct answer: a Rationale: Mixed venous oxygen saturation is affected by four factors: oxygen saturation, hemoglobin, cardiac output, and tissue extraction. In evaluating this patient's decreased SvO2, the nurse has current information for these factors except for cardiac output and tissue extraction. Test-Taking Strategy: Of the given options, option a is the correct choice because ventricular dysrhythmias commonly result in a decrease in cardiac output. If you had no idea of a correct answer, cluster options b, c, and d as noncardiac involvement, then select option a. The clinical situation is related to the heart, so that would be a priority assessment.

A patient arrives in the critical care unit with a diagnosis of acute myocardial infarction. An IV was established in the emergency department. The patient suddenly develops ventricular fibrillation. There is no femoral pulse. What is the priority action? a. Defibrillate with 200 J at once. b. Administer epinephrine. c. Initiate cardiopulmonary resuscitation (CPR). d. Administer amiodarone.

Correct answer: a Rationale: The longer the heart fibrillates, the less likely that defibrillation will be successful. The top priority in ventricular fibrillation is defibrillation. Initial voltage should be 200 J. CPR and epinephrine are indicated after defibrillation is unsuccessful. Amiodarone dosing is initiated after epinephrine. Test-Taking Strategy: Focus on treatment of cause. The patient's cardiac arrest is caused by ventricular fibrillation. Logical treatment would be defibrillation. Though CPR is important in maintaining minimal organ perfusion, it is simply a maintenance mechanism. CPR does not treat the cause of the cardiac arrest. Choose option a. In addition, remember that epinephrine is the number 1 drug in any pulseless situation, but it is not the number 1 treatment for ventricular fibrillation; defibrillation is.

A patient's digital readout of the pulmonary artery pressure suddenly changes from 22/10 mm Hg to 24/2 mm Hg and remains at this pressure. Which of the following is the most likely cause of this change? a. Proximal movement of the catheter b. Distal movement of the catheter c. Hypoxemia d. Pulmonary artery vasodilation

Correct answer: a Rationale: The sudden decrease in the pulmonary artery diastolic pressure (PAd) most often indicates that the catheter has flipped back into the right ventricle. Note that the pulmonary artery systolic pressure has changed little. Distal migration of the catheter would cause the PAOP waveform and pressure. Hypoxemia would increase the PAd as it causes pulmonary hypertension. Pulmonary artery vasodilation would decrease the systolic and diastolic pressures. Two other findings that frequently are associated with proximal migration of the catheter tip back into the right ventricle include loss of dicrotic notch and onset of ventricular ectopy. Test-Taking Strategy: Recall that the systolic pressure of the pulmonary artery and the right ventricle are approximately the same. The diastolic pressure in the pulmonary artery is normally approximately 10 mm Hg, whereas the diastolic pressure of the right ventricle is normally close to 0. Any sudden decrease in the PA diastolic pressure should lead you to suspect that the catheter has flipped back into the right ventricle. Choose option a.

A patient is in cardiac and respiratory arrest. The selection of medications to reestablish cardiac function would stimulate the sympathetic nervous system beta1 receptors. This stimulation would result in increased automaticity and which of the following? a. Increased myocardial contractility b. Decreased left ventricular stroke work c. Decreased myocardial oxygen consumption d. Increased left ventricular afterload

Correct answer: a Rationale: The sympathetic nervous system contains alpha, beta, and dopaminergic receptors that produce various responses when stimulated. Stimulation of beta1 receptors increases heart rate, conductivity, and myocardial contractility. Stimulation of beta2 receptors produces vasodilation and bronchodilation. Alpha receptors, when stimulated, produce vasoconstriction. Stimulation of dopaminergic receptors produces a vasodilating effect on renal, mesenteric, coronary, and cerebral vessels. Test-Taking Strategy: Note the helpful clue in the stem: stimulate the sympathetic system and recall that this results in a fight-or-flight response—the body is responding to survive. Stroke volume would increase, so eliminate option b. Afterload would increase, but this is due to alpha stimulation, not beta1 stimulation, so eliminate option d. Because the heart is working faster and harder, the myocardial oxygen consumption does increase, so eliminate option c. Remember that the primary effects of beta1 receptors are to increase heart rate, contractility, and rate of conduction. Choose option a. A memory aid that also may help is this: beta 1 beta 2, 1 heart 2 lungs: beta1 affects the heart, and beta2 affects the lungs.

A 57-year-old man was admitted to the critical care unit with a diagnosis of anteroseptal myocardial infarction. A pulmonary artery catheter was inserted, and initial readings were within normal limits. Vital signs were blood pressure 140/92 mm Hg, heart rate 110 beats/min and regular, and respiratory rate 24 breaths/min. Breath sounds are equal and clear to auscultation. Three hours after admission, the patient becomes restless with cool, pale skin. Vital signs are now blood pressure 110/72 mm Hg, heart rate 120 beats/min, and respiratory rate 28 breaths/min and labored. Breath sounds are still equal, but crackles are audible at the lung bases bilaterally. The patient is given furosemide (Lasix) at 8 AM. At 9 AM, the pulmonary artery occlusive pressure (PAOP) drops to 8 mm Hg with a drop in the blood pressure. Which of the following would be the most appropriate intervention at this time? a. Administer saline bolus. b. Decrease dobutamine drip rate. c. Increase nitroglycerin drip rate. d. Replace potassium.

Correct answer: a Rationale: Treatment of left ventricular failure requires manipulation of the determinants of stroke volume: preload, afterload, and contractility. Overdiuresis may result in an abnormal reduction of preload that leads to understretch of the myofibrils and decreased contractility. Fluid boluses often are given to restore adequate circulating blood volume and preload and to improve stroke volume and cardiac output. Venous vasodilators are titratable and are better agents than diuretics to reduce preload in a hemodynamically unstable patient. Note that even though a PAOP of 8 mm Hg is within "normal" values, a higher PAOP frequently is required for an optimal myofibril stretch, especially in patients with large ventricular diameter. Test-Taking Strategy: Furosemide decreases preload, and the hemodynamic changes occurred after administration of this drug. Choose an option that would increase preload. Dobutamine increases contractility and decreases preload. Nitroglycerin decreases preload. Potassium would not have a direct effect. Saline bolus would increase preload, so choose option a.

Which of the following describes the timing of the murmur of aortic stenosis? a. Holosystolic b. Systolic ejection c. Early diastolic d. Middiastolic to late diastolic

Correct answer: b Rationale: Aortic stenosis causes a high-pitched, harsh systolic ejection murmur that is loudest at the second right intercostal space (i.e., aortic area) and radiating to the carotid arteries. Mitral regurgitation is a holosystolic murmur. Aortic regurgitation is an early diastolic murmur. Mitral stenosis is a middiastolic to late diastolic murmur. Test-Taking Strategy: Stenosis means that the valve does not open well, so consider the following: When is the aortic valve open? The valve is open during systole, so if the valve does not open well, the turbulence (and the murmur) must occur during systole. Eliminate options c and d. Now, is the answer holosystolic or systolic ejection? Focus further on systole. When does blood go through the aortic valve? The first subphase of systole is isovolumetric contraction when all four valves are closed, no blood is moving, and the pressure in the ventricle is increasing to the point that it exceeds the pressure in the aorta and the aortic valve is opened. The turbulence (and murmur) does not begin until blood starts to go through the aortic valve. Choose systolic ejection, option b.

At which point in the ventilatory cycle should the pulmonary artery occlusive pressure be measured? a. At the peak of inspiration b. At the end of expiration c. The average between inspiration and expiration d. Anytime in the ventilatory cycle

Correct answer: b Rationale: Hemodynamic parameters should be measured at the end of expiration for consistency. If a patient is on a positive pressure volume-cycled mechanical ventilator, inspiration is the high point and expiration is the low or neutral point. If a patient is spontaneously ventilating or is on a pressure-cycled mechanical ventilator, inspiration is negative and expiration is positive. Remember to identify where end expiration is: Patient or Pressure ventilation-Peak, Volume ventilator-Valley. Test-Taking Strategy: Consistency is the most important point here. Remember end expiration. Choose option b.

On a pulmonary artery waveform, the dicrotic notch represents closure of which valve? a. Aortic valve b. Pulmonic valve c. Tricuspid valve d. Mitral valve

Correct answer: b Rationale: In a pulmonary artery waveform the three components of the waveform are systole, dicrotic notch, and diastole. Systole is the pressure generated by the right ventricle so that the pulmonic valve will be pushed open, the dicrotic notch is caused by the closure of the pulmonic valve, and diastole is the pressure in the pulmonary artery during ventricular diastole. The diastolic pressure is a reflection of the vascular tone in the pulmonary vascular bed. If the vessels are constricted or if there is back pressure from the left side of the heart, the diastolic pressure will be high. Test-Taking Strategy: The waveform from the pulmonary artery cannot reflect what is in front of it. Choose the chamber or vessel immediately behind the pulmonary artery. Choose option b. Also, remember that the dicrotic notch on the arterial waveform represents closure of the aortic valve.

Oxygen delivery (DO2) is the product of which of the following? a. PaO2, hemoglobin, mean arterial pressure b. SaO2, hemoglobin, cardiac output c. SvO2, cardiac index, SaO2 d. PaO2, mean arterial pressure, SvO2

Correct answer: b Rationale: Ninety-seven percent of oxygen is attached to the hemoglobin molecule, so the SaO2 (arterial oxygen saturation) is a more accurate reflection of the amount of oxygen in blood. The PaO2 represents only the 3% that is dissolved in the plasma. The lungs must put the oxygen in the blood, the hemoglobin must carry the oxygen, and the cardiac output is a reflection of how well the heart is moving the blood with its hemoglobin with attached oxygen. SvO2 (venous oxygen saturation) is a reflection of the oxygen reserve. SvO2 is what is left over after the tissues have extracted what they need. The mean arterial pressure is a reflection of organ tissue perfusion pressure but does not indicate anything about the amount of oxygen in that blood. Test-Taking Strategy: Oxygen is delivered from the arterial end, so choose an option that has SaO2 instead of SvO2. Also remember that most oxygen is carried on hemoglobin. Look for SaO2 (not PaO2) and hemoglobin. The only option with both of these is option b.

Which of the following is a manifestation of left atrial enlargement on the electrocardiogram? a. Increased amplitude of the P wave on a rhythm strip b. Wide, notched P waves in lead II on 12-lead electrocardiogram c. Diphasic P wave in lead V1 on 12-lead electrocardiogram d. Tall, peaked P waves in lead II on 12-lead electrocardiogram

Correct answer: b Rationale: P waves represent atrial contraction, so look for changes in the P waves as an indication of atrial enlargement. Leads II and V1 are the two best P wave leads. P waves on a voltage standardized 12-lead electrocardiogram should be about 2½ blocks tall and 2½ blocks wide. The P wave in lead II becomes tall and peaked in right atrial enlargement and wide and notched in left atrial enlargement. The P wave is normally diphasic in lead V1. The initial half of the normal diphasic P wave in lead V1 represents right atrial depolarization, and the terminal half of the normal diphasic P wave in lead V1 represents left atrial depolarization. Therefore right atrial enlargement causes a dominant initial half of the diphasic P wave in lead V1, whereas left atrial enlargement causes a dominant terminal half of the diphasic P wave in lead V1. Test-Taking Strategy: Rhythm strips have no standardization for voltage, and enlargement and hypertrophy are manifested by changes in voltage. So eliminate option a because anyone can increase the size of the P wave by increasing the size or gain. Diphasic P waves (ones that are positive and negative) are normal in lead V1, so eliminate option c. Tall, peaked P waves in lead II are called P pulmonale and are associated with right atrial enlargement, so eliminate option d. Wide, notched P waves in lead II are called P mitrale and are associated with left atrial enlargement. Choose option b.

A patient is admitted with acute chest pain and dyspnea. Pulse oximetry indicates an arterial oxygen saturation (SaO2) of 88%. Readings after insertion of the pulmonary artery catheter included a normal pulmonary artery occlusive pressure (PAOP), an elevated pulmonary artery, and an elevated right atrial pressure (RAP). The nurse suspects that these findings are most indicative of what acute problem? a. Cardiac tamponade b. Pulmonary embolism c. Right ventricular infarction d. Pericarditis

Correct answer: b Rationale: Remember that when the pulmonary artery diastolic pressure (PAd) is more than 5 mm Hg greater than the PAOP, pulmonary hypertension is present. The findings are consistent with a problem in the lung that is likely due to a pulmonary embolism. Pericarditis does not affect hemodynamic parameters unless cardiac tamponade occurs. Cardiac tamponade would cause the PAd and PAOP to be elevated and within 5 mm Hg of each other. Elevation and equalization of RAP, PAd, and PAOP is seen in cardiac tamponade and frequently is referred to as equalization of pressure. Right ventricular infarction would cause an elevated right atrial pressure with a normal or low PAOP but would not cause hypoxemia. Test-Taking Strategy: Eliminate answers a and d because they are similar and involve the outside of the heart. Select option b, which is a lung problem.

A patient is admitted to the coronary care unit with third-degree AV heart block, and a transvenous temporary ventricular pacemaker is inserted. Four hours later the patient complains of dizziness while lying in bed. The monitor shows third-degree AV block with a ventricular rate of 52 beats/min and no pacing spikes. This indicates which of the following? a. Failure to capture b. Failure to pace c. Competition between pacemaker and intrinsic rhythm d. Failure to sense

Correct answer: b Rationale: Return of the prepacemaker rhythm indicates a failure to pace. No pacing spikes are evident. Test-Taking Strategy: Failure to capture would be evidenced by pacing spikes not followed by a QRS complex (or P wave if atrial pacer), so eliminate option a. Failure to sense is evidenced by pacing spikes occurring on time regardless of the patient's intrinsic rhythm, so eliminate option d. Failure to sense causes competition between the pacemaker and the intrinsic rhythm, so eliminate option c. Recall that no spikes equals no pacer activity. If there is an intrinsic rate slower than the pacemaker rate, this would indicate a malfunction referred to as failure to pace or failure to fire. Choose b.

Which of the following statements is true? a. The diastolic pulmonary artery pressure (PAd) should not be higher than the pulmonary artery occlusive pressure (PAOP). b. The PAOP should not be higher than the PAd. c. The PAd and the PAOP are usually equal. d. The right atrial pressure (RAP) and the PAOP are usually equal.

Correct answer: b Rationale: The PAd is normally 2 to 5 mm Hg higher than the PAOP. PAd may be more than 5 mm Hg higher than the PAOP in patients with pulmonary hypertension. If the PAOP is higher than the PAd, suspect that there is an occlusion in the catheter or that the catheter is not in the correct area of the pulmonary vasculature. The RAP is normally lower than the PAOP. Test-Taking Strategy: Remember that water does not flow uphill. The circuit between the pulmonary vascular bed and the left atrium is open. The pressure in the pulmonary artery during diastole is the filling pressure for the left atrium. So the PAd should be slightly higher than the PAOP.

Four days after a mitral valve replacement, the patient develops atrial fibrillation. The nurse initially would: a. order a 12-lead electrocardiogram. b. evaluate the patient for clinical indications of hypoperfusion. c. notify the physician. d. ask the patient to bear down as if having a bowel movement.

Correct answer: b Rationale: The onset of atrial fibrillation results in the loss of atrial kick. Loss of atrial kick may reduce the cardiac output by as much as 20-30%. This is especially true in patients whose cardiac output may be affected by long-standing cardiac disease, such as mitral valve disease. Assess the patient for clinical indications of hypoperfusion (e.g., cool skin, decreased urine output, narrowed pulse pressure, and hypotension). Test-Taking Strategy: Remember always being told to treat the patient and not the monitor? What that really means is that clinical assessment is always indicated to evaluate the implications of the rhythm change. Always finish your assessment before notifying the physician so that you have a complete clinical presentation to describe. Eliminate option c. Ordering a 12-lead electrocardiogram is a good option if you cannot interpret the rhythm from the rhythm strip. This question indicates that the rhythm is clear, so the 12-lead electrocardiogram is not the priority action, so eliminate option a. Asking the patient to bear down will increase vagal stimulation. It may slow down but usually will not cause conversion of atrial fibrillation, so eliminate option d. Assessment is the priority in this question. Choose option b.

A 56-year-old man with severe chest pain, diaphoresis, and nausea is brought as a direct admission from the physician's office to the critical care unit by the emergency medical system team. The patient has a patent intravenous line and has been medicated for pain. Upon arrival and before transfer from the stretcher, the patient suddenly becomes apneic and pulseless. The electrocardiogram monitor shows a wide QRS complex rhythm with a regular R-R rhythm and a rate of 160 beats/min. Cardiopulmonary resuscitation is in progress. The cardiac monitor shows a change from a rapid ventricular rhythm to a slow junctional rhythm. What would be the most appropriate next action? a. Assess for spontaneous respirations. b. Assess for a carotid pulse. c. Administer atropine sulfate 1 mg IV. d. Administer epinephrine 1 mg IV.

Correct answer: b Rationale: The patient's pulse always should be checked after each rhythm change to establish whether it is absent or present, in order to determine the next treatment of choice. A slow junctional rhythm with a pulse present may require atropine, depending on the clinical signs and symptoms of the patient. Treatment for pulseless electrical activity would include epinephrine before administration of atropine if it has been at least 3-5 minutes since the last dose of epinephrine was given. Test-Taking Strategy: Apply the nursing process: If the patient has a change of status, do further assessment before an intervention. This narrows the options to a or b; because the change is in the cardiac status, select option b, the assessment of the cardiac system.

A patient with acute anterior myocardial infarction develops a third-degree atrioventricular block with a ventricular escape rhythm with a rate of 38 beats/min. What would be the best immediate intervention? a. Atropine 1 mg b. Transcutaneous pacemaker c. Transvenous pacemaker d. Epinephrine 1 mg

Correct answer: b Rationale: Transcutaneous pacing is the treatment of choice in this situation. Because the left anterior descending artery supplies the bundle of His, this block is located at the level of the bundle of His, which is unresponsive to atropine. Atropine may increase the atrial rate, but it will not increase conduction through the bundle of His because there are no parasympathetic fibers in the intraventricular conduction system. Transcutaneous pacing is followed as soon as possible by transvenous pacing, which provides more reliable capture and less discomfort for the patient. Test-Taking Strategy: Epinephrine is indicated in pulseless situations, and this patient has a pulse, so eliminate option d. Atropine is effective when the problem is in the sinus node, sometimes effective when the problem is in the atrioventricular node, and not effective when the problem is in the bundle of His. Eliminate option a. A transcutaneous pacemaker would be much quicker to implement than a transvenous pacemaker, so choose option b.

While monitoring the patient's pulmonary artery pressure, a damped waveform is noted. Which of the following would not be an appropriate action? a. Ensure that the valve on the balloon lumen is open and that no air is trapped in the balloon. b. Reposition the patient. c. Fast flush the distal lumen. d. Check for a possible clot in the catheter by aspiration of the distal lumen.

Correct answer: c Rationale: A damped pulmonary artery waveform may be caused by air or blood in the pressure monitoring system, a clot in the catheter, or the catheter being advanced distally enough that the catheter diameter occludes the pulmonary arteriole in which the catheter is located (referred to as a spontaneous wedge). First, make sure that air has not been left in the balloon inadvertently by making sure that the balloon lumen is open with the empty syringe attached. Next, search the system for air or blood. Then reposition the patient or ask the patient to cough. If there is no change in the waveform, try to aspirate a clot from the catheter. If there is still no change, have the patient assume a spontaneous wedge position, and the catheter should be repositioned (withdrawn slightly) by the physician as soon as possible to prevent a pulmonary infarction. If the catheter is fast-flushed and a clot is present, it will be embolized with 300 mm Hg pressure (the pressure bag is maintained at this pressure). If the catheter is in a wedge position and the catheter is fast-flushed, it may result in pulmonary arteriole rupture and potentially massive hemoptysis and even exsanguination. Test-Taking Strategy: Remember that this is a negatively stated question. Answers a and b are benign and may help in some situations, so eliminate them. Aspirating from the catheter does break the integrity of the closed circuit but is less dangerous than fast-flushing a catheter that may be occluded (increasing embolus risk) or spontaneously wedged (increasing hemorrhage risk). Option c represents a dangerous action and would not be appropriate, so it is the answer. All other answers are appropriate in this situation.

A 22-year-old patient is admitted to the critical care unit after a motorcycle collision. He has a fractured femur. Shortly after admission, he becomes pulseless, although the monitor shows sinus tachycardia with a rate of 110 beats/min. Which of the following should be included in the patient's initial therapy? a. Intubation, mechanical ventilation, sodium bicarbonate b. Intubation, mechanical ventilation, atropine c. Cardiopulmonary resuscitation (CPR), epinephrine, intravenous fluids d. CPR, dopamine, isoproterenol

Correct answer: c Rationale: A heart rate of 110 beats/min with no pulse is clearly pulseless electrical activity (PEA). Remember that in any pulseless situation, CPR should be initiated as soon as possible (though in ventricular fibrillation, defibrillation is performed first when a defibrillator is available). In addition, in any pulseless situation, the first drug to be given is epinephrine. The next action for PEA is to identify and treat the cause. By history and clinical presentation, you should suspect volume depletion. Test-Taking Strategy: Go with what you know. The patient is pulseless. CPR and epinephrine are your first priorities. This leads you to option c.

A patient becomes apneic and pulseless. Cardiopulmonary resuscitation (CPR) has been initiated, and the monitor shows asystole in two leads. Which of the following drugs would be used initially? a. Calcium gluconate b. Atropine c. Epinephrine d. Amiodarone (Cordarone)

Correct answer: c Rationale: After CPR is initiated and an intravenous access is established, epinephrine should be given. Calcium was used in the past in asystole but is used today only for hypocalcemia, calcium channel blocker toxicity, hyperkalemia, and hypermagnesemia. Atropine is no longer recommended for asystole. Amiodarone is not indicated in asystole because asystole is the absolute absence of irritability. Test-Taking Strategy: Remember that in any pulseless situation, epinephrine is the number 1 drug.

Which of the following types of block are most likely after an anterior wall myocardial infarction (MI)? a. Sinus block b. Second-degree atrioventricular (AV) block, type I c. Second-degree AV block, type II d. Third-degree AV block with junctional escape rhythm

Correct answer: c Rationale: Anterior MI is caused by a left anterior descending (LAD) artery lesion. The LAD artery supplies the bundle of His and bundle branches, so anterior MIs may cause blocks of the bundle of His or bundle branches. Second-degree AV block type II is a block at the level of the bundle of His. If this patient does develop a third-degree AV block, it would be at the level of the bundle of His, and the only escape rhythm available below the bundle of His is a ventricular escape rhythm. Test-Taking Strategy: The sinus node is supplied by right coronary artery (in 55% of people) or left coronary artery (in 45% of people), so eliminate option a. Type I AV block, also called Wenckebach, is a block at the AV node, and the AV node is supplied by right coronary artery (in 90% of people) or left coronary artery (10%), so eliminate option b. The LAD artery supplies the bundle of His, and blocks in this area would eliminate the possibility of junctional escape rhythms, so eliminate option d. Choose option c.

A 68-year-old man with a history of emphysema is scheduled for a colon resection. A pulmonary artery catheter is inserted before surgery and is to be used to guide fluid replacement during and after surgery. Postoperatively his vital signs are blood pressure 104/64 mm Hg, heart rate 116 beats/min, and respiratory rate 32 breaths/min. Which of the following parameters would be most helpful in guiding fluid replacement for this patient? a. Mean arterial pressure (MAP) b. Pulmonary artery diastolic pressure (PAd) c. Pulmonary artery occlusive pressure (PAOP) d. Right atrial pressure (RAP)

Correct answer: c Rationale: Between PAOP measurements, PAd is frequently used to estimate PAOP. However, from this patient's history, you should suspect that this patient has pulmonary hypertension, and his PAd will be considerably higher than the PAOP and not correlate with the PAOP. This means that the PAd will not reflect PAOP and left heart changes in this patient. Only PAOP should be used to estimate left ventricular preload to guide fluid replacement in this patient. Test-Taking Strategy: Remember that fluid equals preload, atrial pressure reflects respective ventricle preload (assuming normal AV valve function and ventricular compliance), and so PAOP (i.e., indirect left atrial pressure) reflects left ventricular preload. Avoid parameters that reflect pulmonary pressures and chambers behind the lungs (e.g., right atrium). Select option c.

A patient is admitted to the coronary care unit with third-degree AV heart block, and a transvenous temporary ventricular pacemaker is inserted. Four hours later the patient complains of dizziness while lying in bed. The monitor shows third-degree AV heart block with a ventricular rate of 52 beats/min and no pacing spikes. This indicates failure to pace, and the patient is taken to surgery for insertion of a DVI permanent pacemaker. Which of the following describes the function of a DVI pacemaker? a. Senses atrium and ventricle; paces ventricle; inhibited by QRS complex b. Senses atrium and ventricle; paces ventricle; triggered by QRS complex c. Paces atrium and ventricle; senses ventricle; inhibited by the QRS complex d. Paces and senses atrium and ventricle; inhibited by the P wave

Correct answer: c Rationale: DVI refers to the North American Society of Pacing and Electrophysiology nomenclature. The first position, D, signifies that the atrium and ventricle are paced; the second position, V, signifies that the ventricle (QRS complex) is sensed; and the third position, I, indicates that the mode of response is inhibited by an intrinsic QRS complex. Test-Taking Strategy: Easy-to-remember tip for pacemaker codes: Think alphabetically. The first position is the chamber(s) paced and the second is the chamber(s) sensed. A is atrium, V is ventricle, and D is dual or both chambers. The only position left is the mode, which can be inhibited (I), triggered (T), or both (D). The most important point to remember in this question is the order of the code. First paced, then sensed, then mode of response.

A 56-year-old man with severe chest pain, diaphoresis, and nausea is brought as a direct admission from the physician's office to the critical care unit by the emergency medical system team. The patient has a patent intravenous line and has been medicated for pain. Upon arrival and before transfer from the stretcher, the patient suddenly becomes apneic and pulseless. The electrocardiogram monitor shows a wide QRS complex rhythm with a regular R-R rhythm and a rate of 160 beats/min. The initial action for this patient is to do which of the following? a. Establish a patent airway with an endotracheal tube. b. Administer epinephrine 1 mg IV push. c. Defibrillate at 200 J. d. Deliver synchronized cardioversion at 100 J.

Correct answer: c Rationale: Early defibrillation is the single most important intervention to improve survival in patients with pulseless ventricular tachycardia or fibrillation. Insertion of the endotracheal tube or administration of epinephrine should be considered only after defibrillation attempts. Nonsynchronized defibrillation should be performed in patients with wide QRS complex tachycardias whose clinical conditions are critically unstable to avoid the delay involved in attempting synchronization with the ventricular rhythm. Test-Taking Strategy: The clues in the stem are the words pulseless, wide QRS complex rhythm, and initial action. With this combination of events, the choice is to defibrillate.

A 57-year-old man was admitted to the critical care unit with a diagnosis of anteroseptal myocardial infarction. A pulmonary artery catheter was inserted, and initial readings were within normal limits. Vital signs were blood pressure 140/92 mm Hg, heart rate 110 beats/min and regular, and respiratory rate 24 breaths/min. Breath sounds are equal and clear to auscultation. Three hours after admission, the patient becomes restless with cool, pale skin. Vital signs are now blood pressure 110/72 mm Hg, heart rate 120 beats/min, and respiratory rate 28 breaths/min and labored. Breath sounds are still equal, but crackles are audible at the lung bases bilaterally. Which medication would reduce this patient's preload most effectively? a. Nitroprusside (Nipride) b. Dopamine HCl (Intropin) c. Nitroglycerin (Tridil) d. Hydralazine HCl (Apresoline)

Correct answer: c Rationale: Hydralazine HCl is an arterial dilator and would decrease afterload (remember arteries are after the heart). Nitroprusside is a mixed vasodilator with predominant arterial end effect. Nitroprusside would decrease afterload more than preload. Dopamine HCl is a vasoconstrictor in dosages greater than 5 mcg/kg/min and increases afterload and possibly preload by decreasing the vascular capacitance. Nitroglycerin dilates predominantly veins (nitroglycerin dilates arteries only if the dosage is at least 1 mcg/kg/min [e.g., greater than 70 mcg/min in a 70-kg patient]). By dilating veins and increasing venule capacitance, venous return to the heart and therefore preload are reduced (remember veins are before the heart). Test-Taking Strategy: Dopamine is an inotropic agent at doses of about 5 mcg/kg/min and primarily a vasoconstrictor at higher doses, so eliminate option b. Hydralazine is an arterial vasodilator, and nitroprusside is a mixed vasodilator with predominantly arterial effects, so eliminate options d and a. Nitroglycerin is a predominantly venous vasodilator. Remember, veins are before (i.e., pre) the heart, and to decrease preload, you must dilate veins. Select option c.

A shift in the point of maximal impulse (PMI) to the fifth left intercostal space at the anterior axillary line could be caused by any of these conditions except: a. left ventricular hypertrophy. b. right tension pneumothorax. c. pericardial effusion. d. right pleural effusion.

Correct answer: c Rationale: In pericardial effusion or tamponade, there is frequently loss of palpability of the apical impulse because of the fluid layer. Left ventricular hypertrophy, right tension pneumothorax, and a right pleural effusion could shift the PMI left of normal. Normally the PMI is located at the fifth left intercostal space at the midclavicular line. Test-Taking Strategy: Look for the process that would not shift the PMI to the left of the normal position. Picture the process. If the left ventricle is big, it will extend farther to the left. If there is excessive pressure or volume on the right side, it will push the heart toward the left. Pericardial effusion will decrease the intensity of the PMI but it won't shift it. Choose option c.

Which vasodilator would be best for a patient with a pulmonary artery occlusive pressure (PAOP) of 24 mm Hg and a systolic vascular resistance (SVR) of 2100 dynes/sec/cm−5? a. Hydralazine (Apresoline) b. Nitroglycerin (Tridil) c. Nitroprusside (Nipride) d. Morphine sulfate

Correct answer: c Rationale: Left ventricular preload (as measured by PAOP) and left ventricular afterload (as measured by SVR) are increased, so venous vasodilation is needed to decrease preload and arterial vasodilation is needed to decrease afterload. Hydralazine dilates arteries only, morphine sulfate dilates veins only, and nitroglycerin dosages must be above 1 mcg/kg/min to achieve arterial dilating effects. Nitroprusside is a mixed vasodilator. It dilates arteries and veins to decrease afterload and preload. Test-Taking Strategy: Take this step by step. The PAOP indicates increased preload. The SVR indicated increased afterload. So both preload and afterload need to be decreased. Veins are pre-heart so dilating veins decreases preload. Arteries are after the heart so dilating arteries decreases afterload. Now which vasodilator dilates both arteries and veins? Of the choices here, nitroprusside is the best choice.

A patient has arrived back in the critical care unit after coronary artery bypass graft. Mediastinal tube drainage is minimal, but the patient is hypotensive with a blood pressure of 80/68 mm Hg. Right atrial pressure (RAP) is 15 mm Hg, pulmonary artery pressure is 35/18 mm Hg, and pulmonary artery occlusive pressure (PAOP) is 17 mm Hg. The PAOP waveform shows large a waves and large v waves. What is the most likely cause of these changes? a. Hemorrhage b. Pulmonary hypertension due to hypoxemia c. Cardiac tamponade d. Acute mitral regurgitation

Correct answer: c Rationale: Note the equalization of pressures. RAP, diastolic pulmonary artery pressure (PAd), and PAOP are elevated and within 5 mm Hg of each other. Large a waves and large v waves (sometimes referred to as M pattern) are seen. The patient is hypotensive. All of these are seen in cardiac tamponade, and cardiac surgery is a major predisposing factor for cardiac tamponade. Hemorrhage would cause the pressures to be decreased. Pulmonary hypertension would cause the PAd to be elevated disproportionately to the PAOP, so PAd would be elevated but PAOP would be normal. Acute mitral regurgitation would cause large v waves but not large a waves. Test-Taking Strategy: Relate what is happening to what has happened. Cardiac surgery can cause option a, b, or c. Option d is much more likely in an acute myocardial infarction, so eliminate option d. Note the minimal mediastinal tube drainage. The expected mediastinal drainage is accumulating around the heart, so choose option c by using a process of elimination based on what you know.

Which of the following hemodynamic parameters is likely to be elevated with right ventricular infarction? a. Systolic pulmonary artery pressure (PAs) b. Pulmonary artery occlusive pressure (PAOP) c. Right atrial pressure (RAP) d. Diastolic pulmonary artery pressure (PAd)

Correct answer: c Rationale: RAP is elevated because of right ventricular failure and back pressure. PAs actually may be decreased because of the inability of the right ventricle effectively to propel blood into the pulmonary artery. PAd and PAOP usually are decreased as a reflection of poor filling caused by decreased right ventricular contractility. The filling of the left side of the heart (preload) is low because of the poor pumping ability of the right side of the heart. Remember that the right side of the heart sends blood to fill the left side of the heart. This is an example of backward failure of the right ventricle and forward failure of the left ventricle. Test-Taking Strategy: Right ventricular infarction frequently causes right ventricular failure. Remember that one of the major signs of right ventricular failure is jugular venous distention, which is a reflection of elevated central venous pressure (CVP). CVP is the pressure in the great vessels as the blood returns to the heart measured from a catheter in the superior vena cava versus RAP, which is the pressure in the right atrium measured from the proximal port of a pulmonary artery catheter. Although these are not exactly the same in actuality, they are the same in practicality. The terms frequently are used interchangeably. Choose option c.

A 57-year-old man was admitted to the critical care unit with a diagnosis of anteroseptal myocardial infarction. A pulmonary artery catheter was inserted, and initial readings were within normal limits. Vital signs were blood pressure 140/92 mm Hg, heart rate 110 beats/min and regular, and respiratory rate 24 breaths/min. Breath sounds are equal and clear to auscultation. Three hours after admission, the patient becomes restless with cool, pale skin. Vital signs are now blood pressure 110/72 mm Hg, heart rate 120 beats/min, and respiratory rate 28 breaths/min and labored. Breath sounds are still equal, but crackles are audible at the lung bases bilaterally. Which of the following readings would indicate an increased left ventricular preload for this patient? a. Pulmonary artery systolic pressure (PAs) 30 mm Hg b. Pulmonary artery diastolic pressure (PAd) 12 mm Hg c. Pulmonary artery occlusive pressure (PAOP) 25 mm Hg d. Right atrial pressure (RAP) 13 mm Hg

Correct answer: c Rationale: Remember that the atrial pressure is reflective of the end-diastolic pressure for the respective ventricle in the absence of atrioventricular valve disease. So RAP is reflective of right ventricular preload, and left atrial pressure (LAP) is reflective of left ventricular preload. PAOP is an indirect measurement of LAP and therefore left ventricular end-diastolic pressure. Normal PAOP is 8 to 12 mm Hg. Left ventricular failure and/or fluid overload leads to an increase in the PAOP. Test-Taking Strategy: RAP is reflective of right ventricular preload. Consider PAs as reflective of the right ventricle because it is the pressure generated by the right ventricle to open the pulmonic valve; consider PAd as reflective of the pulmonary vasculature because it is the pressure in the pulmonary artery, whereas the right ventricle is in the relaxation phase and blood is leaving the large pulmonary arteries and entering arterioles and capillaries; consider the PAOP as reflective of the left ventricle because pressures of the right side of the heart are blocked by the inflated balloon. The only parameter among these three that relates to the left side of the heart is the PAOP.

A 25-year-old semicomatose woman is admitted to the critical care unit. The patient's friend reports the patient's use of illegal drugs. General assessment reveals a cachectic body. A prolonged QT segment is noted on the 12-lead electrocardiogram. Which electrolyte is least likely to be the cause of the observed electrical changes? a. Potassium b. Calcium c. Sodium d. Magnesium

Correct answer: c Rationale: Sodium affects phase 0 of the action potential or the cellular response to a stimulus that results in depolarization. A prolonged QT segment represents prolonged repolarization. Calcium, magnesium, or potassium imbalances may affect phases 2 and 3 of the action potential by shortening or lengthening this phase. Low levels of any of these would result in prolonged repolarization, which would be seen as a prolonged QT segment. Test-Taking Strategy: If you have no idea of the correct answer, use a common sense approach. Potassium, calcium, and magnesium abnormalities almost always are seen together. Also, abnormal levels are most likely to result in more severe abnormal body functions. Abnormal sodium levels are less likely to have as dramatic findings unless levels are extremely abnormal, as may be seen in endocrine abnormalities.

A patient is admitted to the coronary care unit with third-degree AV block, and a transvenous temporary ventricular pacemaker is inserted. Four hours later the patient complains of dizziness while lying in bed. The monitor shows third-degree AV heart block with a ventricular rate of 52 beats/min and no pacing spikes. This indicates failure to pace. Based on this interpretation, which is the most appropriate first action? a. Increase the sensitivity. b. Increase the milliamperage. c. Check the electrode connections. d. Replace the generator batteries.

Correct answer: c Rationale: The malfunction of the pacemaker is probably due to a technical malfunction. Checking the electrode lead connections would be the best initial action. Next, change the batteries or increase the milliamperage. Increasing the sensitivity would be inappropriate because there does not appear to be a failure to sense, which would be evidenced by pacing spikes occurring on time regardless of the intrinsic rhythm. Test-Taking Strategy: Use the nursing process; do further assessment before interventions. Options a, b, and d are interventions. Choose option c.

A patient with chest pain, dizziness, and shortness of breath is admitted to the critical care unit. The monitor shows type II second-degree AV block with a rate of 40 beats/min. The initial treatment for this patient's heart rhythm is which of the following actions? a. Administration of epinephrine 1 mg IV b. Administration of adenosine (Adenocard) 6 mg IV c. Application of transcutaneous pacemaker d. Preparation of the patient immediately for insertion of an external transvenous pacemaker

Correct answer: c Rationale: Transcutaneous pacing is the initial treatment of choice because of the speed with which it can be instituted. Transvenous pacing is indicated as soon as possible because of the potential discomfort of transcutaneous pacing. Adenosine is given in supraventricular tachycardias and not slow rhythms, and epinephrine would be used for asystole but not if the patient has a rhythm and cardiac output. You may have considered atropine (though it was not listed as an option), but it would have been contraindicated because of the chest pain and, therefore, presumed myocardial ischemia because it could cause significant tachycardia and an increase in myocardial oxygen consumption. Test-Taking Strategy: Eliminate option b because adenosine is used for supraventricular tachycardias and not for bradycardias. Epinephrine is used in pulseless situations, so eliminate option a. This patient does need a pacemaker, so choose the one immediately available. Choose option c.

When pulmonary arterial diastolic pressure (PAd) is more than 5 mm Hg higher than pulmonary artery occlusive pressure (PAOP), it signals which abnormal condition? a. Right ventricular failure b. Left ventricular failure c. Pulmonary hypertension d. Systemic hypertension

Correct answer: c Rationale: When the PAd is more than 5 mm Hg higher than the PAOP, it is an indication of pulmonary hypertension. Possible causes of pulmonary hypertension are passive (e.g., mitral valve disease) or active (e.g., causes of hypoxemic pulmonary vasoconstriction such as acute respiratory distress syndrome, chronic obstructive pulmonary disease, or pulmonary embolism). Pulmonary embolism causes pulmonary hypertension by mechanical obstruction and by hypoxemic pulmonary vasoconstriction. Test-Taking Strategy: Consider the pulmonary artery systolic pressure as reflective of the right side of the heart, the PAd as reflective of the pulmonary vascular circuit, and the PAOP as reflective of the left side of the heart. Therefore the PAd would be reflective of a pulmonary vascular circuit problem. Left ventricular failure increases the PAd, but the PAOP is elevated also and there is not more than a 5-mm Hg difference between the PAd and the PAOP. Right ventricular failure increases the RAP. Systemic hypertension increases systemic pressures.

1A 52-year-old man has undergone coronary artery bypass grafting and has returned to the critical care unit. During the last hour his parameters were as follows: O2 saturation 99% PaO2 with FiO2 0.4 184 mm Hg Hemoglobin 12 gm/dL Serum potassium 3.8 mEq/L Urine output 100 mL/hr Now the patient's venous oxygen saturation (SvO2) monitor shows a rapid decline from a reading of 75% to 62% without any change in arterial oxygen saturation (SaO2). His cardiac rhythm changes from normal sinus rhythm to a sinus rhythm with ventricular bigeminy. The nurse should first: a. check serum arterial blood gases. b. administer 10 mEq potassium. c. increase the FiO2. d. check the serum potassium level.

Correct answer: d Rationale: Because the SaO2 has not changed, the drop in SvO2 most likely is linked to the decrease in cardiac output caused by the ventricular ectopy. The nurse's initial action would be to identify the cause so that treatment can be directed toward that cause. Given the patient's urine output of 100 mL/hr, hypokalemia is a likely cause as a result of the high urine output because, in most cases, potassium excretion is related directly to urine output. Test-Taking Strategy: Note that the assessment of arterial blood gases (option a) and administration of additional oxygen (option c) seem to be good answers, but the SaO2 has not changed. This indicates that the decrease in SvO2 is due to a decrease in cardiac output or hemoglobin or an increase in oxygen consumption at the tissue level. The last recorded hemoglobin was acceptable, and there is no new evidence of bleeding. Because of the recent onset of dysrhythmia, an additional drop in potassium should be suspected but should be confirmed, so select option d.

A patient arrived in the emergency department with complaints of chest pain. The 12-lead electrocardiogram shows ST segment elevation in leads V3 and V4. Occlusion of the affected coronary artery most likely would affect perfusion to which portion of the conduction system? a. Sinoatrial (SA) node b. Bachmann's bundle c. Atrioventricular (AV) node d. Bundle of His

Correct answer: d Rationale: ST segment elevation in leads V3 and V4 indicates injury to the anterior wall, which would occur with occlusion of the left anterior descending (LAD) artery. In most persons, the SA node, Bachmann's bundle, and AV node are supplied by the right coronary artery. The bundle of His is supplied by the left anterior descending artery. This is why an anterior myocardial infarction may cause type II second-degree AV block or third-degree AV heart block at the level of the bundle of His. Test-Taking Strategy: Note that options a, b, and c are part of the supraventricular conduction system. They usually are supplied by the right coronary artery. The LAD artery supplies most of the interventricular conduction system, including the bundle of His and the bundle branches.

A patient develops atrial fibrillation after abdominal surgery. Her blood pressure falls from 110/70 mm Hg to 92/68 mm Hg. The hypotension is related to which of the following? a. Decrease in ventricular contractility b. Hypovolemia c. Mural thrombi d. Decrease in ventricular filling

Correct answer: d Rationale: The contribution that atrial contraction makes to ventricular filling volume is approximately 15% to 30%. Atrial fibrillation results in quivering but not contracting atria. The loss of 15% to 30% of diastolic filling volume reduces cardiac output and can have significant hemodynamic consequences. Although mural thrombi also are a problem, they result in an embolic phenomenon rather than a direct decrease in cardiac output. The relationship between the development of atrial fibrillation and the decrease in cardiac output make hypovolemia and decrease in contractility less likely. Test-Taking Strategy: Relate recent changes in patient status to recent occurrences. The patient had a change in atrial function, so select an option that results in loss of atrial contraction or "kick." Choose option d.

A woman, age 35 years, reports the feeling that her heart is racing out of her chest, shortness of breath, and dizziness on admission to the critical care unit. The patient reports a history of a "floppy valve" for the past 10 years. Which of the following is true regarding the murmur of mitral valve prolapse? a. Early systolic with a low-pitched, blowing quality b. Radiates to the carotid arteries c. Loudest at the lower left sternal border d. Usually accompanied by a midsystolic click

Correct answer: d Rationale: The murmur of mitral valve prolapse is caused by mitral regurgitation. Mitral regurgitation murmurs are high-pitched, blowing, systolic murmurs that are loudest at the apex and radiate to the axilla. When specific to mitral valve prolapse, a midsystolic click usually is heard at the apex, and the murmur follows the click such as S1, click, murmur, S2. Test-Taking Strategy: Remember that all murmurs are high-pitched with the exceptions of mitral stenosis and tricuspid stenosis, so eliminate option a. Radiation is in the direction of blood flow. In this case, the direction is from the left ventricle to the left atrium, which causes the radiation to the axilla, not the carotid arteries. Eliminate option b. Mitral regurgitation murmurs are loudest at the apex, also called the mitral area. Eliminate option c. The click is associated with a mitral valve leaflet bulging toward the left atrium during midsystole. Remember that mitral valve prolapse also is called click-murmur syndrome. Choose option d.

A 70-year-old woman, weighing 50 kg, comes to the emergency department complaining of chest pain and shortness of breath. The electrocardiogram monitor shows ventricular tachycardia at a rate of 150 beats/min. Which treatment is appropriate in this situation? a. Amiodarone (Cordarone) IV b. Verapamil HCl (Calan) IV c. Defibrillation beginning at 200 J d. Synchronized cardioversion beginning at 100 J

Correct answer: d Rationale: Treatment for an unstable ventricular tachycardia is sedation and synchronized cardioversion beginning at 100 J. Amiodarone would be used in stable ventricular tachycardia. Verapamil is used for supraventricular tachycardias and would be inappropriate for ventricular tachycardia. Defibrillation is indicated if the patient is pulseless. Test-Taking Strategy: Go with what you know. The patient is conscious, so cardioversion is more appropriate than defibrillation. Choose option d.

What is the most frequently identified primary mechanism of cardiac arrest? a. Asystole b. Ventricular tachycardia c. Third-degree atrioventricular block d. Ventricular fibrillation

Correct answer: d Rationale: Ventricular fibrillation is the most frequently identified primary mechanism of cardiac arrest. Test-Taking Strategy: The key word is primary. Cardiac arrest most likely is caused by irritability. Options b and d are examples of ventricular irritability. Ventricular tachycardia does not always cause cardiac arrest, but ventricular fibrillation does. Choose option d.


Set pelajaran terkait

ACCTG 101 - Chapter 7 - Smart learning

View Set

Inflammation, Infection, Nutrition

View Set

Iowa Laws, Rules, and Regulations Common to All Lines

View Set

CH 1 K01001 History of the automobile

View Set

Knowledge of the principles and practices related to long term systematic atmospheric monitoring of pollutants in ambient air.

View Set